3

Question How do the Bell numbers grow? Specifically if $b(n)$ denotes the $n$-th Bell number then which function $g(n)$ satisfies $b(n)=O(g(n))$ for $n\to \infty$?

My Attempt

At first i tried to come up with some usefull boundaries. A rather obvious one presented to be that for all $n\in \mathbb{N}$, $b(n)\leq n!$. I did prove this via induction over $n$. In the following ${n\brace k}$ denotes the stirling numbers of the second kind.

$ \underline{n=0} $ $$ b(0)=1\leq 0!=1 \implies \text{ True} $$ $ \underline{n\implies n+1} $ $$ \begin{equation*} \begin{split} b(n+1)&\leq (n+1)!\\ \iff \hspace{73pt} \sum\limits_{k=0}^{n+1}{n+1\brace k}&\leq(n+1)!\\ \iff \qquad \underbrace{\sum\limits_{k=0}^{n+1}{n\brace k-1}}_{b(n)}+\sum\limits_{k=0}^{n+1}k{n\brace k}&\leq (n+1)!\\ \overset{(n+1)!=nn!+n!}{\iff} \hspace{35pt} \underbrace{\sum\limits_{k=0}^{n+1}\frac{k}{n}{n\brace k}}_{{n\brace n+1}=0/\frac{k}{n}\leq 1\implies \leq b(n)}&\leq n!\implies\text{ True} \end{split} \end{equation*} $$ So the Bell numbers grow at most factorially

Now Iam going to use the following function for the Bell numbers $b(n)=\frac{1}{e}\sum_{r=0}^{\infty}\frac{r^n}{r!}$.

Let $q\in\mathbb{R}_+$ and $C>0$, then there exists a $N\in\mathbb{N}$ such that for all $n\geq N$

$$ \begin{equation*} \begin{split} \frac{b(n)}{q^n}=\frac{1}{e}\sum\limits_{r=0}^{\infty}\left(\frac{r}{q}\right)^n\frac{1}{r!}&=\frac{1}{e}\biggl[\sum\limits_{r=0}^{\lceil q+1 \rceil}\left(\frac{r}{q}\right)^n\frac{1}{r!}+\underbrace{\sum\limits_{r=\lceil q+2\rceil}^{\infty}\left(\frac{r}{q}\right)^n\frac{1}{r!}}_{\geq 0}\biggr]\\ &\geq \frac{1}{e}\sum\limits_{r=0}^{\lceil q+1\rceil}\left(\frac{r}{q}\right)^n\frac{1}{r!}\geq \frac{1}{e(\lceil q+1\rceil)!}\left(\frac{\lceil q+1\rceil}{q}\right)^n\geq C \end{split} \end{equation*} $$ This shows that the Bell numbers grow faster than any exponential function of the form $q^n$. In particular it follows that $e^n=o(b(n))$ for $n\to\infty$. So the Bell numbers grow faster than $e^n$ but not faster than $n!$, but how exactly they grow I wasnt able to determine. Your help is much appreciated.

Emar
  • 420
  • 2
    Why didn't you mention the results in the Wikipedia entry of the Bell numbers? – jjagmath Mar 09 '25 at 20:35
  • @jjagmath, why should I? – Emar Mar 09 '25 at 20:38
  • 1
    To show that you have investigated on your own about the problem, instead of expecting others to make that for you. – jjagmath Mar 09 '25 at 20:43
  • @jjagmath, well I clearly investigated the problem. What you see is how far I got. To add to that wikipedia doesnt really give much context around the given asymptotic expressions. Altrought there are some books linked, I would not have asked if I wouldnt liked for someone to put these expressions in context. Overall I think linking wikipedia and similar sources seems rather unnecessary. – Emar Mar 09 '25 at 20:54
  • 3
    The exact answer is given on Wikipedia, although in a somewhat unfriendly way (https://en.wikipedia.org/wiki/Bell_number#Growth_rate). It can be deduced using saddle points. – Qiaochu Yuan Mar 09 '25 at 21:21
  • 2
    $B_n\le n!$ because of the obvious surjection from the set of all permutations of a finite set $E$ to the set of all partitions of $E$. – user14111 Mar 10 '25 at 07:05
  • 3
    The very recent paper https://arxiv.org/pdf/2408.14182 gives simple upper and lower bounds – Claude Leibovici Mar 10 '25 at 07:20

2 Answers2

5

If you look at sequence $A000110$ in $OEIS$, you will find a very asymptotics for Bell numbers (it was proposed in year 2002 by Benoit Cloitre and corrected by Vaclav Kotesovecin year 2013.

It write $$B_n=b^n e^{b-n-\frac{1}{2}}\sqrt{\frac{b}{b+n}}\qquad \text{with}\qquad b=e^{W\left(n-\frac{1}{2}\right)}$$ where $W(.)$ is the main branch of Lambert function.

I think that, as a closed form, this one is better than what is given in the linked Wikipedia. The absolute relative error is smaller than $0.05$% for $n=10$.

Using the series expansion of Lambert function, then the formula proposed by de Bruijn in year 1981 given in the Wikipedia page.

3

To get the precise asymptotic growth is not so easy and is usually done using the saddle point method. To get the asymptotic growth up to a factor of $n$, with upper and lower bounds, can be done in a relatively low-tech elementary way as follows. First, a lower bound:

Lower bound: We have $$B_n \ge \frac{k^n}{k!}$$ for every positive integer $k$.

Note that this is slightly better (by a factor of $e$) than what we get from picking a single term in Dobinski's formula.

Proof. By Burnside's lemma, $\frac{k^n}{k!}$ is a lower bound on the number of orbits of the action of $S_k$ on the set of functions $[n] \to [k]$, where $[n] = \{ 1, 2, \dots n \}$. These orbits correspond to partitions of $n$ into at most $k$ non-empty subsets, so there are at most $B_n$ of them. $\Box$

For fixed $k$ this already shows (without Dobinski's formula) that $B_n$ grows faster than exponential. But the real significance of this bound is that we can optimize $k$ as a function of $n$. We consider the function $f(k) = \frac{k^n}{k!}$ and compute the effect of increasing $k$ by $1$, namely

$$\frac{f(k+1)}{f(k)} = \frac{(k+1)^n}{k^n (k+1)} = \frac{\left( 1 + \frac{1}{k} \right)^n}{k+1}.$$

We want $k = k_n$ to be the smallest positive integer such that this ratio is $\le 1$, so that increasing $k$ no longer improves the bound. Equivalently, if $x_n$ denotes the exact real solution to the equation

$$x_n + 1 = \left( 1 + \frac{1}{x_n} \right)^n$$

then $k_n = \lceil x_n \rceil$. Applying the trusty exponential inequality $(1 + x)^n \le e^{nx}$ to the RHS gives

$$x_n \le x_n + 1 \le \exp \left( \frac{n}{x_n} \right) \Rightarrow$$ $$\frac{n}{x_n} \exp \left( \frac{n}{x_n} \right) \ge n.$$

The equation $we^w = n$ has solution $w = W(n)$ where $W$ is the Lambert $W$ function; this is, as far as I know, the simplest argument which explains why the Lambert $W$ function appears in the asymptotics of the Bell numbers. $\frac{n}{x_n}$ is slightly bigger than this, which gives

$$\begin{align*} \frac{n}{x_n} &\ge W(n) \Leftrightarrow \\ x_n &\le \frac{n}{W(n)} \Rightarrow \\ k_n &\le \boxed{ \left\lceil \frac{n}{W(n)} \right\rceil }. \end{align*}$$

We can prove a matching lower bound on $k_n$ but we don't really need it; using this choice for $k$ is enough, and gives:

Corollary: We have $$\boxed{ B_n \ge \frac{\left\lceil \frac{n}{W(n)} \right\rceil^n}{\left\lceil \frac{n}{W(n)} \right\rceil!} }.$$

This lower bound has the correct asymptotic growth up to a factor of $O(\sqrt{n})$ or so. You can get something that looks more like the other bounds on Wikipedia or in Claude's answer and comment using Stirling's formula. I like this bound because it has a nice conceptual interpretation: heuristically speaking, what it's saying is that a random set partition resembles the set partition obtained from the fibers of a random function $[n] \to [k]$ where $k \approx \frac{n}{W(n)}$. This suggests the following, all of which are true:

  • the expected number of parts of a random set partition is roughly $\frac{n}{W(n)}$,
  • the expected size of a random part of a random set partition is roughly $W(n)$,
  • the expected number of parts of size $k$ has roughly a Poisson shape $\frac{W(n)^k}{k!}$.

In turn, one way to motivate the significance of the number $k \approx \frac{n}{W(n)}$ here is that it inverts the coupon collection time! It is in fact the solution to $n = k \log k$, so it tells us when a random function $[n] \to [k]$ is likely to be surjective, hence when the corresponding partition of $[n]$ is likely to not contain empty subsets.

There is a matching (up to a factor of $O(n)$) upper bound which can be derived by a similar argument, but we need to reason about the decomposition $B_n = \sum S(n, k)$ of the Bell numbers into the Stirling numbers of the second kind; I will edit it in later.


Edit: Here is a sketch of the upper bound. We use:

Upper bound: We have $$S(n, k) \le \frac{k^n}{k!}$$ for every $1 \le k \le n$.

Proof. $S(n, k) k!$ is the number of surjective functions $[n] \to [k]$, which is at most the number of all functions. $\Box$

Corollary: We have $$B_n \le \sum_{k=1}^n \frac{k^n}{k!}.$$

Note the similarity to Dobinski's formula, but without the factor of $e^{-1}$, and we get to truncate to $n$ terms. Now we can apply the following simple idea: just bound this sum by $n$ times its largest term. We already defined this largest term to occur at $k = k_n$, so we get:

Corollary: We have $$\boxed{ B_n \le n \frac{k_n^n}{k_n!} }$$ where $k_n$ maximizes $\frac{k^n}{k!}$.

This is within exactly a factor of $n$ from our lower bound, which was actually $B_n \ge \frac{k_n^n}{k_n!}$. From here we finally need a lower bound on $k_n$ to locate it more precisely; I believe but have not carefully checked that it is always either equal to $\left\lceil \frac{n}{W(n)} \right\rceil$ or $\left\lfloor \frac{n}{W(n)} \right\rfloor$.

This means either our lower or our upper bound must be correct to within a factor of $\sqrt{n}$. A natural guess is that we should take the geometric mean of our bounds to get a better estimate, giving

$$B_n \approx \sqrt{n} \frac{k_n^n}{k_n!}$$

which we can test the accuracy of numerically. For $n = 10$ we have

$$B_{10} = \boxed{ 115975 }$$ $$x_{10} = \frac{10}{W(10)} = 5.73 \dots $$ $$k_{10} = 6$$ $$\frac{6^{10}}{6!} = 83980.8$$ $$10 \frac{6^{10}}{6!} = 839808$$ $$\sqrt{10} \frac{6^{10}}{6!} = 265570.6 \dots $$

which is not bad but suggests that a better estimate, taking the hint from Dobinski's formula, would be to divide by $e$; this gives

$$e^{-1} \sqrt{10} \frac{6^{10}}{6!} = \boxed{ 97698.0 \dots }$$

and suggests (without proof) the estimate

$$\boxed{ B_n \approx e^{-1} \sqrt{n} \frac{ \left( \frac{n}{W(n)} \right)^n}{ \left( \frac{n}{W(n)} \right)!} }$$

which I believe (but have not checked carefully) is correct asymptotically or close to it, possibly off by a small constant factor. Comparing to the other known asymptotics can be done, again, using Stirling's.

I think we should be able to recover the correct square root factor rigorously from Dobinski's formula by estimating the contributions of the terms other than the dominant term $k = k_n$ more closely; they should end up looking approximately like a Gaussian times the dominant term, so we'll get a factor that looks like a Gaussian integral.

Qiaochu Yuan
  • 468,795
  • Is it possible to give a asymptotic formula without the W function? – Beren Gunsolus Mar 15 '25 at 16:35
  • 1
    @Beren: it's a natural question, but as far as I know the answer is no. The issue is that while it's technically possible to describe the asymptotics of $W(n)$ in an elementary way (we have $W(n) = \ln n - \ln \ln n + o(1)$ although the error term here decays quite slowly so this is misleading), the Bell numbers require the asymptotics of $W(n)^n$, which as far as I know is not elementary at all. After working with these estimates for awhile I believe the Lambert W function is intrinsic to the situation and is fundamental enough that one should just regard it as an elementary function. – Qiaochu Yuan Mar 15 '25 at 16:51
  • Thanks! In your experience do the bell numbers feel the most natural sequence that doesn't have an elementary growth rate? Also, if there's any hope we could prove bell growth isn't elementary without lambert w? – Beren Gunsolus Mar 15 '25 at 18:55
  • 1
    @Beren: yes, I actually asked a version of this question about elementary growth rates here: https://mathoverflow.net/questions/376140/are-there-natural-sequences-with-exotic-growth-rates-what-metatheorems-are I don't know how to prove the growth rate isn't elementary at all but presumably it will involve the true growth rate, hence the Lambert W function. There is really no reason to try to avoid it, it is just what naturally appears here (e.g. as above it naturally appears as describing both the mean and modal number of parts of a random set partition). – Qiaochu Yuan Mar 15 '25 at 19:14